www.vorhilfe.de
- Förderverein -
Der Förderverein.

Gemeinnütziger Verein zur Finanzierung des Projekts Vorhilfe.de.
Hallo Gast!einloggen | registrieren ]
Startseite · Mitglieder · Impressum
Forenbaum
^ Forenbaum
Status VH e.V.
  Status Vereinsforum

Gezeigt werden alle Foren bis zur Tiefe 2

Navigation
 Startseite...
 Suchen
 Impressum
Das Projekt
Server und Internetanbindung werden durch Spenden finanziert.
Organisiert wird das Projekt von unserem Koordinatorenteam.
Hunderte Mitglieder helfen ehrenamtlich in unseren moderierten Foren.
Anbieter der Seite ist der gemeinnützige Verein "Vorhilfe.de e.V.".
Partnerseiten
Weitere Fächer:

Open Source FunktionenplotterFunkyPlot: Kostenloser und quelloffener Funktionenplotter für Linux und andere Betriebssysteme
Forum "Rationale Funktionen" - Kurvendiskussion
Kurvendiskussion < Rationale Funktionen < Analysis < Oberstufe < Schule < Mathe < Vorhilfe
Ansicht: [ geschachtelt ] | ^ Forum "Rationale Funktionen"  | ^^ Alle Foren  | ^ Forenbaum  | Materialien

Kurvendiskussion: Frage (beantwortet)
Status: (Frage) beantwortet Status 
Datum: 15:22 So 09.03.2008
Autor: Markus110

Aufgabe
Geg. f(x)= [mm] \bruch{x^2-2x+1}{x+1} [/mm]

Zeigen Sie das der Graph keine Wendepunkte (geändert: nicht -tangente!!) besitzt! Berechnen Sie die Stellen x, an denen die Funktion den Anstieg -3 hat!

[winken] Hallo Zusammen!

[mm] D_f= \IR [/mm]  /(-1)

Habe alle Werte berechnet: Nullstelle (1;0); Schnittpunkt y-Achse (0;1); Minimum bei (1;0); Maximum bei (-3;-8). Die Ableitungen auch

f'(x)= [mm] \bruch{x^2+2x-3}{(x+1)^2} [/mm]  und f''(x)= [mm] \bruch{10x+8}{(x+1)^4} [/mm]

Nun zu meinen Fragen:  Der Zähler der zweiten Ableitung wird Null bei x= [mm] -\bruch{4}{5} [/mm] ; ist das der

Wendepunkt? Und wie beweise ich, dass die Funktion keine Wendepunkte (!!) besitzt?

Beim Anstieg m= -3, reicht es den Zähler der ersten Ableitung -3 zu setzen?

Also [mm] x^2+2x-3=-3 [/mm] <=> x(x-2)=0 => [mm] x_1= [/mm] 0 und [mm] x_2= [/mm] -2

Oder muss ich den Nenner mit einbeziehen? Brauch ich den Nenner überhaupt zur Kurvendiskussion, ausser zur Definitionsbereichsbestimmung und Polfindung?

Ist einfach zu lange her bei mir.....Danke schonmal für Eure Hilfe. LG Markus


        
Bezug
Kurvendiskussion: Antwort
Status: (Antwort) fertig Status 
Datum: 15:41 So 09.03.2008
Autor: angela.h.b.


> Geg. f(x)= [mm]\bruch{x^2-2x+1}{x+1}[/mm]
>
> Zeigen Sie das der Graph keine Wendetangente besitzt!
> Berechnen Sie die Stellen x, an denen die Funktion den
> Anstieg -3 hat!
>  [winken] Hallo Zusammen!
>  
> [mm]D_f= \IR[/mm]  /(-1)
>  
> Habe alle Werte berechnet: Nullstelle (1;0); Schnittpunkt
> y-Achse (0;1); Minimum bei (1;0); Maximum bei (-3;-8). Die
> Ableitungen auch
>  
> f'(x)= [mm]\bruch{x^2+2x-3}{(x+1)^2}[/mm]  und f''(x)=
> [mm]\bruch{10x+8}{(x+1)^4}[/mm]
>  
> Nun zu meinen Fragen:  Der Zähler der zweiten Ableitung
> wird Null bei x= [mm]-\bruch{4}{5}[/mm] ;

Hallo,

Du solltest Deine 2. Ableitung nochmal rechnen. Ich bekomme da nämlich etwas anderes - was auch besser zur Aufgabenstellung paßt.

Gruß v. Angela

Bezug
                
Bezug
Kurvendiskussion: Frage (beantwortet)
Status: (Frage) beantwortet Status 
Datum: 16:24 So 09.03.2008
Autor: Markus110

Aufgabe
s.o.


Habe noch 2x nachgerechnet und für f''(x)= [mm] \bruch{8x+8}{(x+1)^4} [/mm] raus.  

Immer wieder der fehlerteufel.... stimmt das? also wäre x=-1 für 8x+8=0
Und wie geht das mit den Wendepunkten? Danke erstmal.

LG Markus

Bezug
                        
Bezug
Kurvendiskussion: Antwort
Status: (Antwort) fertig Status 
Datum: 16:31 So 09.03.2008
Autor: angela.h.b.


> s.o.
>  
> Habe noch 2x nachgerechnet und für f''(x)=
> [mm]\bruch{8x+8}{(x+1)^4}[/mm] raus.  

Hm - ich hatte [mm] \bruch{4x+4}{(x+1)^4} [/mm] ausgerechnet, aber das muß nicht unbedingt etwas zu bedeuten haben...

Jedenfalls ist sowohl in Deiner als auch in meiner Variante die einzige Nullstelle der zweiten Ableitung bei x=-1, dh. nur an dieser Stelle könnte es einen Wendepunkt geben.
x=-1 hat allerdings den Nachteil, daß die Funktion hier überhaupt nicht definiert ist...
Und wo's keinen Wendepunkt gibt, gibt's auch keine Wendetangente.

Gruß v. Angela


Bezug
                                
Bezug
Kurvendiskussion: Korrektur
Status: (Frage) beantwortet Status 
Datum: 16:37 So 09.03.2008
Autor: Markus110

Aufgabe
Anstieg m=-3

Hallo Angela!

Danke, stimmt für -1 is ja nicht definiert. Hab ich sonst richtig gerechnet für die Punkte wo der Anstieg -3 sein soll? Und reicht der Zähler um die Funktion zu diskutieren? Danke nochmal. LG Markus

Bezug
                                        
Bezug
Kurvendiskussion: Antwort
Status: (Antwort) fertig Status 
Datum: 16:47 So 09.03.2008
Autor: angela.h.b.

Hallo,

für die Stelle, wo die Steigung -3 ist, mußt Du die 1. Ableitung =-3 setzen,

also  $ [mm] \bruch{x^2+2x-3}{(x+1)^2} [/mm] $ =-3 lösen.

Nicht nur den Zähler! Das mit dem Zähler gilt nur, wenn man Nullstellen sucht, weil ja ein Bruch nur =0 sein kann, wenn sein Zähler =0 ist.

Gruß v. Angela

Bezug
                                                
Bezug
Kurvendiskussion: Korrektur
Status: (Frage) beantwortet Status 
Datum: 20:34 So 09.03.2008
Autor: Markus110

Aufgabe
Anstieg= -3

Ich hoffe, dass ich richtig gerechnet habe.

habe: $ [mm] \bruch{x^2+2x-3}{(x+1)^2} [/mm] $ =-3  


Also im einzelnen:  [mm] \bruch{x^2+2x-3}{(x+1)^2} [/mm] =-3     [mm] /*(x+1)^2 [/mm]

                    [mm] x^2+2x-3 [/mm] = [mm] -3(x+1)^2 [/mm]
                    [mm] x^2+2x-3 [/mm] = [mm] -3(x^2+2x+1) [/mm]
                    [mm] x^2+2x-3 [/mm] = [mm] -3x^2-6x-3 [/mm]       /-2x  [mm] /-x^2 [/mm]
                          -3 = [mm] -4x^2-8x-3 [/mm]     /+3
                           0 = [mm] -4x^2-8x [/mm]

                          [mm] x_1= [/mm] 0 ; [mm] x_2= [/mm] -2
                      
          Passt das? Danke. LG Markus

Bezug
                                                        
Bezug
Kurvendiskussion: Antwort
Status: (Antwort) fertig Status 
Datum: 20:38 So 09.03.2008
Autor: Tyskie84

Hallo [winken]> Anstieg= -3
>  Ich hoffe, dass ich richtig gerechnet habe.
>  
> habe: [mm]\bruch{x^2+2x-3}{(x+1)^2}[/mm] =-3  
>
>
> Also im einzelnen:  [mm]\bruch{x^2+2x-3}{(x+1)^2}[/mm] =-3    
> [mm]/*(x+1)^2[/mm]
>  
> [mm]x^2+2x-3[/mm] = [mm]-3(x+1)^2[/mm]
>                      [mm]x^2+2x-3[/mm] = [mm]-3(x^2+2x+1)[/mm]
>                      [mm]x^2+2x-3[/mm] = [mm]-3x^2-6x-3[/mm]       /-2x  
> [mm]/-x^2[/mm]
>                            -3 = [mm]-4x^2-8x-3[/mm]     /+3
>                             0 = [mm]-4x^2-8x[/mm]
>
> [mm]x_1=[/mm] 0 ; [mm]x_2=[/mm] -2
>                        
> Passt das? Danke. LG Markus

Ja es passt alles [daumenhoch]

[cap] Gruß


Bezug
Ansicht: [ geschachtelt ] | ^ Forum "Rationale Funktionen"  | ^^ Alle Foren  | ^ Forenbaum  | Materialien


^ Seitenanfang ^
ev.vorhilfe.de
[ Startseite | Mitglieder | Impressum ]